Question on point set topologyDefinition of Borel setsA “complementary” topologyFinite vs infinite...

How to terminate ping <dest> &

Help rendering a complicated sum/product formula

How does 取材で訪れた integrate into this sentence?

Would it be believable to defy demographics in a story?

What does "^L" mean in C?

Can you move over difficult terrain with only 5 feet of movement?

Using Past-Perfect interchangeably with the Past Continuous

World War I as a war of liberals against authoritarians?

Help prove this basic trig identity please!

In Aliens, how many people were on LV-426 before the Marines arrived​?

Have the tides ever turned twice on any open problem?

Unfrosted light bulb

What (if any) is the reason to buy in small local stores?

Relation between independence and correlation of uniform random variables

What is the relationship between relativity and the Doppler effect?

Variable completely messes up echoed string

How is the partial sum of a geometric sequence calculated?

How do hiring committees for research positions view getting "scooped"?

Can a wizard cast a spell during their first turn of combat if they initiated combat by releasing a readied spell?

gerund and noun applications

I seem to dance, I am not a dancer. Who am I?

Are dual Irish/British citizens bound by the 90/180 day rule when travelling in the EU after Brexit?

Pronounciation of the combination "st" in spanish accents

Why is indicated airspeed rather than ground speed used during the takeoff roll?



Question on point set topology


Definition of Borel setsA “complementary” topologyFinite vs infinite distinction in Rudin's AnalysisThe set of rationals in $(0,1)$ is not a $G_delta$Limit point of an infinite subset of a compact setIf $U ⊂ mathbb{R}^n$ is open and $B ⊂ U$, then why is it that $B$ relatively open in $U$ if and only if $B$ is open?Question about Theorem 2.24 in Baby RudinShowing that if closed subsets don't intersect then there exists open sets in which they exist that also don't intersectDifference between closure and closed cover of a setIs there an analogue for a compact set using closed sets?













1












$begingroup$


Does there exist a closed set which is an intersection of a collection of infinite open sets?










share|cite|improve this question







New contributor




Tony Tong is a new contributor to this site. Take care in asking for clarification, commenting, and answering.
Check out our Code of Conduct.







$endgroup$








  • 5




    $begingroup$
    Intersect $(-tfrac{1}{n}, tfrac{1}{n})$ for $n = 1, 2, ldots$ and consider what set you get
    $endgroup$
    – Brevan Ellefsen
    20 mins ago










  • $begingroup$
    Oh it will get ${0}$
    $endgroup$
    – Tony Tong
    15 mins ago
















1












$begingroup$


Does there exist a closed set which is an intersection of a collection of infinite open sets?










share|cite|improve this question







New contributor




Tony Tong is a new contributor to this site. Take care in asking for clarification, commenting, and answering.
Check out our Code of Conduct.







$endgroup$








  • 5




    $begingroup$
    Intersect $(-tfrac{1}{n}, tfrac{1}{n})$ for $n = 1, 2, ldots$ and consider what set you get
    $endgroup$
    – Brevan Ellefsen
    20 mins ago










  • $begingroup$
    Oh it will get ${0}$
    $endgroup$
    – Tony Tong
    15 mins ago














1












1








1





$begingroup$


Does there exist a closed set which is an intersection of a collection of infinite open sets?










share|cite|improve this question







New contributor




Tony Tong is a new contributor to this site. Take care in asking for clarification, commenting, and answering.
Check out our Code of Conduct.







$endgroup$




Does there exist a closed set which is an intersection of a collection of infinite open sets?







analysis






share|cite|improve this question







New contributor




Tony Tong is a new contributor to this site. Take care in asking for clarification, commenting, and answering.
Check out our Code of Conduct.











share|cite|improve this question







New contributor




Tony Tong is a new contributor to this site. Take care in asking for clarification, commenting, and answering.
Check out our Code of Conduct.









share|cite|improve this question




share|cite|improve this question






New contributor




Tony Tong is a new contributor to this site. Take care in asking for clarification, commenting, and answering.
Check out our Code of Conduct.









asked 23 mins ago









Tony TongTony Tong

292




292




New contributor




Tony Tong is a new contributor to this site. Take care in asking for clarification, commenting, and answering.
Check out our Code of Conduct.





New contributor





Tony Tong is a new contributor to this site. Take care in asking for clarification, commenting, and answering.
Check out our Code of Conduct.






Tony Tong is a new contributor to this site. Take care in asking for clarification, commenting, and answering.
Check out our Code of Conduct.








  • 5




    $begingroup$
    Intersect $(-tfrac{1}{n}, tfrac{1}{n})$ for $n = 1, 2, ldots$ and consider what set you get
    $endgroup$
    – Brevan Ellefsen
    20 mins ago










  • $begingroup$
    Oh it will get ${0}$
    $endgroup$
    – Tony Tong
    15 mins ago














  • 5




    $begingroup$
    Intersect $(-tfrac{1}{n}, tfrac{1}{n})$ for $n = 1, 2, ldots$ and consider what set you get
    $endgroup$
    – Brevan Ellefsen
    20 mins ago










  • $begingroup$
    Oh it will get ${0}$
    $endgroup$
    – Tony Tong
    15 mins ago








5




5




$begingroup$
Intersect $(-tfrac{1}{n}, tfrac{1}{n})$ for $n = 1, 2, ldots$ and consider what set you get
$endgroup$
– Brevan Ellefsen
20 mins ago




$begingroup$
Intersect $(-tfrac{1}{n}, tfrac{1}{n})$ for $n = 1, 2, ldots$ and consider what set you get
$endgroup$
– Brevan Ellefsen
20 mins ago












$begingroup$
Oh it will get ${0}$
$endgroup$
– Tony Tong
15 mins ago




$begingroup$
Oh it will get ${0}$
$endgroup$
– Tony Tong
15 mins ago










1 Answer
1






active

oldest

votes


















4












$begingroup$

$$mathbb{R}capmathbb{R}capmathbb{R}capcdots$$






share|cite|improve this answer









$endgroup$













  • $begingroup$
    But R is an open set, the intersection is also R so it is still an open set
    $endgroup$
    – Tony Tong
    19 mins ago








  • 1




    $begingroup$
    And also closed
    $endgroup$
    – Keen-ameteur
    17 mins ago






  • 1




    $begingroup$
    While extremely simple, this example has the unfortunate side effect of also being open, which could further confound the OP who seems to be wondering why the intersection of open sets is not open in general (admittedly, the OP is probably also drawing a false dichotomy between open and closed sets, so I suppose this helps with that)
    $endgroup$
    – Brevan Ellefsen
    13 mins ago












  • $begingroup$
    @BrevanEllefsen: +1, but I couldn't resist... :-)
    $endgroup$
    – parsiad
    13 mins ago













Your Answer





StackExchange.ifUsing("editor", function () {
return StackExchange.using("mathjaxEditing", function () {
StackExchange.MarkdownEditor.creationCallbacks.add(function (editor, postfix) {
StackExchange.mathjaxEditing.prepareWmdForMathJax(editor, postfix, [["$", "$"], ["\\(","\\)"]]);
});
});
}, "mathjax-editing");

StackExchange.ready(function() {
var channelOptions = {
tags: "".split(" "),
id: "69"
};
initTagRenderer("".split(" "), "".split(" "), channelOptions);

StackExchange.using("externalEditor", function() {
// Have to fire editor after snippets, if snippets enabled
if (StackExchange.settings.snippets.snippetsEnabled) {
StackExchange.using("snippets", function() {
createEditor();
});
}
else {
createEditor();
}
});

function createEditor() {
StackExchange.prepareEditor({
heartbeatType: 'answer',
autoActivateHeartbeat: false,
convertImagesToLinks: true,
noModals: true,
showLowRepImageUploadWarning: true,
reputationToPostImages: 10,
bindNavPrevention: true,
postfix: "",
imageUploader: {
brandingHtml: "Powered by u003ca class="icon-imgur-white" href="https://imgur.com/"u003eu003c/au003e",
contentPolicyHtml: "User contributions licensed under u003ca href="https://creativecommons.org/licenses/by-sa/3.0/"u003ecc by-sa 3.0 with attribution requiredu003c/au003e u003ca href="https://stackoverflow.com/legal/content-policy"u003e(content policy)u003c/au003e",
allowUrls: true
},
noCode: true, onDemand: true,
discardSelector: ".discard-answer"
,immediatelyShowMarkdownHelp:true
});


}
});






Tony Tong is a new contributor. Be nice, and check out our Code of Conduct.










draft saved

draft discarded


















StackExchange.ready(
function () {
StackExchange.openid.initPostLogin('.new-post-login', 'https%3a%2f%2fmath.stackexchange.com%2fquestions%2f3152433%2fquestion-on-point-set-topology%23new-answer', 'question_page');
}
);

Post as a guest















Required, but never shown

























1 Answer
1






active

oldest

votes








1 Answer
1






active

oldest

votes









active

oldest

votes






active

oldest

votes









4












$begingroup$

$$mathbb{R}capmathbb{R}capmathbb{R}capcdots$$






share|cite|improve this answer









$endgroup$













  • $begingroup$
    But R is an open set, the intersection is also R so it is still an open set
    $endgroup$
    – Tony Tong
    19 mins ago








  • 1




    $begingroup$
    And also closed
    $endgroup$
    – Keen-ameteur
    17 mins ago






  • 1




    $begingroup$
    While extremely simple, this example has the unfortunate side effect of also being open, which could further confound the OP who seems to be wondering why the intersection of open sets is not open in general (admittedly, the OP is probably also drawing a false dichotomy between open and closed sets, so I suppose this helps with that)
    $endgroup$
    – Brevan Ellefsen
    13 mins ago












  • $begingroup$
    @BrevanEllefsen: +1, but I couldn't resist... :-)
    $endgroup$
    – parsiad
    13 mins ago


















4












$begingroup$

$$mathbb{R}capmathbb{R}capmathbb{R}capcdots$$






share|cite|improve this answer









$endgroup$













  • $begingroup$
    But R is an open set, the intersection is also R so it is still an open set
    $endgroup$
    – Tony Tong
    19 mins ago








  • 1




    $begingroup$
    And also closed
    $endgroup$
    – Keen-ameteur
    17 mins ago






  • 1




    $begingroup$
    While extremely simple, this example has the unfortunate side effect of also being open, which could further confound the OP who seems to be wondering why the intersection of open sets is not open in general (admittedly, the OP is probably also drawing a false dichotomy between open and closed sets, so I suppose this helps with that)
    $endgroup$
    – Brevan Ellefsen
    13 mins ago












  • $begingroup$
    @BrevanEllefsen: +1, but I couldn't resist... :-)
    $endgroup$
    – parsiad
    13 mins ago
















4












4








4





$begingroup$

$$mathbb{R}capmathbb{R}capmathbb{R}capcdots$$






share|cite|improve this answer









$endgroup$



$$mathbb{R}capmathbb{R}capmathbb{R}capcdots$$







share|cite|improve this answer












share|cite|improve this answer



share|cite|improve this answer










answered 20 mins ago









parsiadparsiad

18.4k32453




18.4k32453












  • $begingroup$
    But R is an open set, the intersection is also R so it is still an open set
    $endgroup$
    – Tony Tong
    19 mins ago








  • 1




    $begingroup$
    And also closed
    $endgroup$
    – Keen-ameteur
    17 mins ago






  • 1




    $begingroup$
    While extremely simple, this example has the unfortunate side effect of also being open, which could further confound the OP who seems to be wondering why the intersection of open sets is not open in general (admittedly, the OP is probably also drawing a false dichotomy between open and closed sets, so I suppose this helps with that)
    $endgroup$
    – Brevan Ellefsen
    13 mins ago












  • $begingroup$
    @BrevanEllefsen: +1, but I couldn't resist... :-)
    $endgroup$
    – parsiad
    13 mins ago




















  • $begingroup$
    But R is an open set, the intersection is also R so it is still an open set
    $endgroup$
    – Tony Tong
    19 mins ago








  • 1




    $begingroup$
    And also closed
    $endgroup$
    – Keen-ameteur
    17 mins ago






  • 1




    $begingroup$
    While extremely simple, this example has the unfortunate side effect of also being open, which could further confound the OP who seems to be wondering why the intersection of open sets is not open in general (admittedly, the OP is probably also drawing a false dichotomy between open and closed sets, so I suppose this helps with that)
    $endgroup$
    – Brevan Ellefsen
    13 mins ago












  • $begingroup$
    @BrevanEllefsen: +1, but I couldn't resist... :-)
    $endgroup$
    – parsiad
    13 mins ago


















$begingroup$
But R is an open set, the intersection is also R so it is still an open set
$endgroup$
– Tony Tong
19 mins ago






$begingroup$
But R is an open set, the intersection is also R so it is still an open set
$endgroup$
– Tony Tong
19 mins ago






1




1




$begingroup$
And also closed
$endgroup$
– Keen-ameteur
17 mins ago




$begingroup$
And also closed
$endgroup$
– Keen-ameteur
17 mins ago




1




1




$begingroup$
While extremely simple, this example has the unfortunate side effect of also being open, which could further confound the OP who seems to be wondering why the intersection of open sets is not open in general (admittedly, the OP is probably also drawing a false dichotomy between open and closed sets, so I suppose this helps with that)
$endgroup$
– Brevan Ellefsen
13 mins ago






$begingroup$
While extremely simple, this example has the unfortunate side effect of also being open, which could further confound the OP who seems to be wondering why the intersection of open sets is not open in general (admittedly, the OP is probably also drawing a false dichotomy between open and closed sets, so I suppose this helps with that)
$endgroup$
– Brevan Ellefsen
13 mins ago














$begingroup$
@BrevanEllefsen: +1, but I couldn't resist... :-)
$endgroup$
– parsiad
13 mins ago






$begingroup$
@BrevanEllefsen: +1, but I couldn't resist... :-)
$endgroup$
– parsiad
13 mins ago












Tony Tong is a new contributor. Be nice, and check out our Code of Conduct.










draft saved

draft discarded


















Tony Tong is a new contributor. Be nice, and check out our Code of Conduct.













Tony Tong is a new contributor. Be nice, and check out our Code of Conduct.












Tony Tong is a new contributor. Be nice, and check out our Code of Conduct.
















Thanks for contributing an answer to Mathematics Stack Exchange!


  • Please be sure to answer the question. Provide details and share your research!

But avoid



  • Asking for help, clarification, or responding to other answers.

  • Making statements based on opinion; back them up with references or personal experience.


Use MathJax to format equations. MathJax reference.


To learn more, see our tips on writing great answers.




draft saved


draft discarded














StackExchange.ready(
function () {
StackExchange.openid.initPostLogin('.new-post-login', 'https%3a%2f%2fmath.stackexchange.com%2fquestions%2f3152433%2fquestion-on-point-set-topology%23new-answer', 'question_page');
}
);

Post as a guest















Required, but never shown





















































Required, but never shown














Required, but never shown












Required, but never shown







Required, but never shown

































Required, but never shown














Required, but never shown












Required, but never shown







Required, but never shown







Popular posts from this blog

Paper upload error, “Upload failed: The top margin is 0.715 in on page 3, which is below the required...

Emraan Hashmi Filmografia | Linki zewnętrzne | Menu nawigacyjneGulshan GroverGulshan...

How can I write this formula?newline and italics added with leqWhy does widehat behave differently if I...